When Mr. Jackson got in his car yesterday, the odometer read 187,198.9 km. When he got home, the reading was 187,399.4 km. How far did Mr. Jackson drive?

Answers

Answer 1

The distance Mr. Jackson drove from when he got in his car to when he got home is 200.5 km.

Given:

The odometer reading when Mr.Jackson got in his car = 187,198.9 km

The odometer reading when Mr.Jackson got home = 187,399.4 km

An odometer is a device used to calculate the distance traveled by a vehicle.

To determine the distance Mr.Jackson drove we subtract the odometer readings from when he got home minus when he got into his car.

⇒ (187,399.4 - 187,198.9) km

⇒ 200.5 km

Therefore, Mr. Jackson drove 220.5 km from when he got in his car to when he got home.

To learn more about distance visit:

https://brainly.com/question/27033538

#SPJ10


Related Questions

4. Manu determines the roots of a polynomial equation by applying the theorems he knows. He organizes the results of these theorems.
From the fundamental theorem of algebra, Manu knows there are 3 roots to the equation.
From Descartes’ rule of sign, Manu finds no sign changes in and 3 sign changes in .
The rational root theorem yields as the list of possible rational roots.
The lower bound of the polynomial is .
The upper bound of the polynomial is 1.
What values in Manu’s list of rational roots should he try in synthetic division in light of these findings?

Answers

The values in Manu’s list of rational roots he should try in the synthetic division, in light of these findings is

"Manu should try first +1/4,+1/2 and +1."

This is further explained below.

What is the synthetic division?

Generally, Because the lower and higher bonds are located between -6 and 1, Manu should have only analyzed the potential rational zeros that fall between those two numbers.

A lower bond indicates that the feasible rational zero cannot be lower than -6, and therefore the higher number that may be achieved cannot be more than 1.

In this manner, Manu will get an opportunity to test. This approach of rationalization based on bonds aims to reduce the number of potential solutions.

Therefore, if Manu discovers through the synthetic division that the possible roots are +1/4,+1/2,+1,+5/4,+2,+5/2,+4,+5,+10,+20, then he should only consider those inside the intervals marked by the lower and upper bounds, which are +1/4,+1/2,+1, because the rest is greater than 1. This is because the rest of the possible roots are higher than 1.

Read more about synthetic division

https://brainly.com/question/28824872

#SPJ1

CQ

Manu determines the roots of a polynomial equation p(x)=0 by applying the theorems he knows. He organizes the results of these theorems. From the fundamental theorems of algebra, Manu knows there are 3 roots to the equation. From Descartes' rule of sign, Manu finds no sign changes in p(x) and 3 sign changes in p(-x). The rational root theorem yields +1/4,+1/2,+1,+5/4,+2,+5/2,+4,+5,+10,+20 as a list of possible rational roots. The lower bound of the polynomial is -6. The upper bound of the polynomial is 1. What values in Manu's list of rational roots should he try in synthetic division in light of these findings?

Solve the system of inequalities by graphing.
y ≥ 2
y < 4
Select a line to change it between solid and dotted. Select a region to shade it.

Answers

The area between the solid line (y=x+4) and the dotted line (y=-2x-2) represents the system of inequalities solution set.

Two linear inequalities system on a coordinate plane. The first has a solid line graphed with a negative slope of one, a negative y-intercept, and a shaded origin area. The area encompassing the origin is shaded, and the second is a dashed vertical line 3 units to the left of the origin.

x ≥ –3; y ≥ x – 2

x > –3; 5y ≥ –4x – 10

x > –3; y ≥ –x + 1

x > –2; y ≥ –x – 1

The solution set for the system of inequalities is represented by the region between the solid line (y=x+4) and the dotted line (y=-2x-2).

Learn more about graphs at

https://brainly.com/question/7149541

#SPJ9

Answer the questions below.

Answers

(a) Amount of data entered

The amount of data entered affects his pay at the end of the week.

(b) R

The independent variable is the input.

(c) Average temperature

The average temperature depends on the city's distance from the equator.

The graph shows the reciprocal parent function.Which statement best describes the function?O A. The function is negative when x < 0.OB. The function is never negative.C. The function is negative when x > 0.O D. The function is negative when x < 0 and also when x > 0.PREVIOUS

Answers

Let's begin by identifying key information given to us in the graph:

The reciprocal parent function is given as 1/f(x). This is better written as shown below

[tex]\begin{gathered} f(x)=\frac{a}{(x-h)}+k \\ when\colon h=0,k=0,a=1 \\ f(x)=\frac{1}{x-0}+0 \\ f(x)=\frac{1}{x} \\ f(x)=y \\ \Rightarrow y=\frac{1}{x} \end{gathered}[/tex]

When the value for x is greater than zero, the function is positive as shown below:

[tex]\begin{gathered} x=2 \\ y=\frac{1}{2}=\frac{1}{2} \\ y=\frac{1}{2} \end{gathered}[/tex]

When the value of x is lesser than zero, the function is negative as shown below:

[tex]\begin{gathered} x=-1 \\ y=\frac{1}{-1}=-1 \\ y=-1 \end{gathered}[/tex]

Therefore, the correct answer is option A (The function is negative when x < 0)

20 students started the class. Then 2 students dropped the
class.

What percent of the students have dropped the class?

Answers

If 20 was at the beginning and 2 students drop the class it would be 10%

Trying to find the length of the robot arm given the xy point along with angle theta and angle theta r

Answers

8a. The length of the robot arm, and the angle in radian and degrees are as follows

length of the robot arm = 30 unitsangle in radian = 7π/6 radian (anticlockwise)angle in degrees = 210 degrees (anticlockwise)

8b. The length of the robot arm, and the angle in radian and degrees are as follows

length of the robot arm = 60 unitsangle in radian = π/6 radian (clockwise)angle in degrees = 210 degrees (clockwise)How to find the length of the robot arm and the angles

8a

i. let the length of the robot arm be r

The length of the lever arm using the points is done by

r^2 = ( -26 )^2 + ( -15 )^2

r^2 = 676 + 225

r^2 = 900

r = √ 900

r = 30

ii. theta in radian

tan θ = opposite / adjacent

tan θ = y direction / x direction

tan θ = -15 / -26

tan θ = 15/26

θ = Arc tan ( 15/26 )

θ = 29.9816 degrees

θ ≈ 30 degrees

θ ≈ 30 degrees + 180 = 210 degrees

In radian

π = 180 degrees

? = 210 degrees

we cross multiply to get

? * 180 degrees =  π * 210 degrees

? =  π * 210 degrees / 180 degrees

= 7π/6 radian

8b

i. let the length of the lever arm be r

The length of the lever arm using the points is done by

r^2 = ( -30 )^2 + ( 52 )^2

r^2 = 900 + 2704

r^2 = 3604

r = √ 3604

r = 60.033

r ≈ 60

ii. theta in radian

tan θ = opposite / adjacent

tan θ = y direction / x direction

tan θ = -30 / 52

tan θ = -30/52

θ = Arc tan ( -30/52 )

θ = -29.9816 degrees

θ ≈ -30 degrees (clockwise direction)

In radian

π = 180 degrees

? = 30 degrees

we cross multiply to get

? * 180 degrees =  π * 30 degrees

? =  π * 30 degrees / 180 degrees

= π/6 radian

Read more on angles here: https://brainly.com/question/25770607

#SPJ1

-3-23x=-14(x-21)-15(x+33)
Find (x)

Please I keep messing up somewhere so please show step by step

Answers

The value of (x) that satisfy the equation → - 3 - 23x = - 14(x - 21) - 15(x + 33) is x = - 33.

What is Equation?

An equation is a mathematical statement with an 'equal to' symbol between two expressions that have equal values.

Given is the following equation -

- 3 - 23x = - 14(x - 21) - 15(x + 33)

The given equation is -

- 3 - 23x = - 14(x - 21) - 15(x + 33)

Simplifying for (x), we get -

- 3 - 23x = - 14x + 294 - 15x - 495

- 23x + 14x + 15x = 3 + 294 - 495

6x = - 198

x = - 33

Therefore, the value of (x) that satisfy the equation → - 3 - 23x = - 14(x - 21) - 15(x + 33) is x = - 33.

To solve more questions on Simplifying equations, visit the link below-

https://brainly.com/question/26127254

#SPJ1

equati 1. v = 3.1 + 14 y = -1

Answers

y= 3x +14 (a)

y=-4x (b)

Replace the value of y (b) on equation (a), and solve for x

-4x = 3x+14

-4x-3x = 14

-7x = 14

x= 14/-7

x = -2

Replace the value of x on any initial equation and solve for y:

y= -4 (-2) = 8

y=8

Answer:

1*v=3,0.1+14y=-1 : v=3,y=-0.07857

[1.v=3}

[0.1+14y=-1]

v=3,y=-0.07857

Step-by-step explanation:

Find sin(2x), cos(2x), and tan(2x) from the given information.
tan(x) = − 4/3 , x in Quadrant II

Answers

Answer:

Step-by-step explanation:

Starting from tan x, you can find sec x, because of the trigonometric identity 1 + tan2 x = sec2 x.

1 + (-4/3)2 = 1 + 16/9 = 25/9 = sec2 x.

So sec x = ±√(25/9) = ±5/3.  But since x is in Quadrant II, sec x has to be negative.  That's because sec x has the same sign as cos x, because sec x = 1 / cos x.  We know that cos x is negative is Quadrant II, therefore so is sec x.  So sec x = -5/3.

Since sec x and cos x are reciprocals of each other, cos x = 1/sec x = -3/5.

Now use the identity sin2 x + cos2 x = 1, to find sin x:

sin2 x + (-3/5)2 = 1

sin2 x + 9/25 = 1

sin2 x = 16/25

sin x = ±4/5

Again, we know that sin x is positive in Quadrant II, so sin x = 4/5.

Now that we know sin x and cos x, we can use the double angle formulas to find sin 2x and cos 2x.

sin 2x = 2 sin x cos x = 2 (4/5) (-3/5) = -24/25

cos 2x = cos2 x - sin 2 x = (-3/5)2 - (4/5)2 = 9/25 - 16/25 = -7/25

Finally use the identity tan x = sin x / cos x to find tan 2x:

tan 2x = sin 2x / cos 2x = (-24/25) / (-7/25) = -24/-7 = 24/7

10a-4a simplify
PLEASEE I NEED HELLPPP

Answers

they answer is 6a because you mines 10-4 and you get 6 then just put the loke 6a

(3)/(4)[(-15+4)+(6+7)-:(-3)]

Answers

Answer:

The answer is -23/2 or -11.5

Step-by-step explanation:

What is the slope of the line passing through (-2, 4)
and (3, -4)?

Answers

Answer:

[tex]\frac{-8}{5}[/tex]

Step-by-step explanation:

Slope is the change in y over the change in x

[tex]\frac{y_{2 - y_{1} } }{x_{2 - y_{1} } }[/tex]

(-2,4) is ([tex]x_{1}[/tex], [tex]y_{1}[/tex])

(3. -4) is ([tex]x_{2}[/tex],[tex]y_{2}[/tex])

[tex]\frac{-4 - 4}{3 - -2}[/tex] = [tex]\frac{-8}{3+ 2}[/tex] = [tex]\frac{-8}{5}[/tex]

Answer:

-8 / 5

Step-by-step explanation:

We can use the slope formula

m = ( y2-y1)/(x2-x1)

    = ( -4 -4)/( 3 - -2)

   = -8/ ( 3+2)

   = -8 / 5

During a basketball game, you made 11 shots of either 2 or 3 points. You scored atotal of 25 points. How many shots of each point value did you make?

Answers

During a basketball game, you made 11 shots of either 2 or 3 points. You scored a

total of 25 points. How many shots of each point value did you make?

Let

x -----> number of shots of 2 points

y -----> number of shots of 3 points

we have that

x+y=11

x=11-y -----> equation A

and

2x+3y=25 -----> equation B

substitute equation A in equation B

2(11-y)+3y=25

solve for y

22-2y+3y=25

y=25-22

y=3

Find the value of x

x=11-3

x=8

therefore

number of shots of 2 points is 8number of shots of 3 points is 3

Select all the intervals where if is increasing

Answers

The given function is increasing over these following intervals:

B. -2 ≤ x ≤ -1.

C. 0 < x < 1.

Where to find where a function is increasing, from it's graph?

Considering the graph of the function, we get that a function f(x) is increasing when it is "moving northeast", that is, to the right and up on the graph, meaning that when x increases, y increases.Conversely, we get that a function f(x) is decreasing when it is "moving southeast", that is, to the right and down the graph, meaning that when x increases, y decreases.

Considering the given definitions on the bullet points above, the behavior of the function can be separated as follows:

Increasing on -2.5 ≤ x ≤ 2.5.Decreasing on all other intervals except the one above.

Items B and C have subsets of the interval -2.5 ≤ x ≤ 2.5, hence the function is increasing on those intervals, and they are the correct options for this problem.

More can be learned about increasing functions at https://brainly.com/question/1503051

#SPJ1

Which of the following is the correct mathematical expression for:

The difference between three times a number and 4

A. 3x + 4
B. 3x - 4
C. 1/3x + 4
D. 1/3x - 4

Answers

Answer:

B

Step-by-step explanation:

if the number is x then 3 times the number is 3x

the difference is the subtraction of 3x and 4 , that is

3x - 4

Question 2-22
A cake is cut into 12 equal slices. After 3 days Jake has eaten 5 slices. What is his weekly rate of eatin
5
36
35
36
O

cakes/week
1
01 cakes/week
35
1
1 cakes/week
4
< Previous
cakes/week
2022-2023 T-Math-Gr7Reg-T2-CBT: Section 2-...

Answers

Jake's weekly rate of eating of cake slices is 11.6

Given,

Number of equal slices of cake = 12

Number of slices Jake eaten after 3 days = 5

We have to find the weekly rate of eating;

Here,

Number of days in a week = 7

Jake eaten 5 slices in 3 days so, 7 - 3 = 4

Again after 3 days 5 slices.

Then,

4 - 3 = 1

That is, Jake eaten 10 slices of cake in 6 days.

Number of cake slices eaten in 1 day = 5/3 = 1.6

Therefore,

Weekly rate of eating is 10 + 1.6 = 11.6

That is,

Jake's weekly rate of eating of cake slices is 11.6

Learn more about rate of eating here;

https://brainly.com/question/28953060

#SPJ1

Use y = 3x2 + 18x - 2 to answer the following question(1, 19) is a point on the graph. What point is the reflection of (1, 19) across the axis of symmetry of the parabola?

Answers

Since y is a parabolla, there will be two values for y = 19. We already know that x = 1 is one value, to find the other, we can substitute y = 19 on the equation and solve for x to get the following:

[tex]\begin{gathered} 19=3x^2+18x-2 \\ \Rightarrow3x^2+18x-2-19=0 \\ \Rightarrow3x^2+18x-21=0 \\ \Rightarrow3(x^2+6x-7)=0 \\ \Rightarrow3(x-1)(x+7)=0 \end{gathered}[/tex]

the solutions of the equation are x = 1 and x = -7. Since we already have that (1,19) is a point on the graph, then we have that the other point is (-7,19)

The National Opinion Research Center administered the General Social Surve
persons in the United States who were 18 years of age or older. One question asked
respondents for their highest grade of school completed (Respondent's Years of Education).
Another question asked respondents for the highest grade of school that their father had
completed (Father's Years of Education).
The equation for the least squares regression line for predicting a respondent's years of
education, ý, from the years of education for the respondent's father, x, is:
ý = 9.996 + 0.355x
Predict the years of education for a person whose father had 14 years of education. Round to
the nearest whole number.

Answers

The years of education as per the given equation, for a person whose father had 14 years of education is 14.966 years.

What is education?

A planned activity, education has objectives like knowledge transmission or character and skill development. The development of understanding, reason, kindness, and honesty may be some of these goals.

As given in the question,

respondent's years of education is represented by y, and

respondent's father years of education is represented by x,

The equation for the least squares regression line for predicting a respondent's years of education is given as:

y = 9.996 + 0.355x

The year of education of respondent's father is given as 14

So putting the value of x in the given equation as 14:

we get,

y = 9.996 + 0.355(14)

y = 14.966

Hence, the years of education for a person whose father had 14 years of education is 14.966 years.

To know more about education, go to link

https://brainly.com/question/25887038

#SPJ9

a box contains 3 white balls and 4 black balls. a ball is drawn at random the color is recorded and then the ball is put back in the box. Then a second ball is drawn at random from the same box. find the probability of the event that at least one of the balls is white

Answers

The box has 3 white balls and 4 black balls.

Total number of balls = 3 + 4 = 7

First draw:

The probability of getting a white ball is given by

[tex]\begin{gathered} P(white)=\frac{\text{number of white balls}}{total\text{ number of balls}} \\ P(white)=\frac{3}{7} \end{gathered}[/tex]

Second draw:

Notice that after the first draw the ball is put back in the box.

The probability of getting a white ball is given by

[tex]P(white)=\frac{3}{7}[/tex]

At least one of the balls is white means that one white ball or two white balls.

[tex]P(x\ge1)\; =P(x=1)+P(x=2)_{}[/tex]

We have already found the probability of getting one white ball that is P(x=1) = 3/7

The probability of getting two white balls is

[tex]\begin{gathered} P(two\; white)=P(white)\times P(white) \\ P(two\; white)=\frac{3}{7}\times\frac{3}{7}=\frac{9}{49} \end{gathered}[/tex]

Finally, the probability of at least one white ball is

[tex]\begin{gathered} P(x\ge1)\; =P(x=1)+P(x=2)_{} \\ P(x\ge1)\; =\frac{3}{7}+\frac{9}{49} \\ P(x\ge1)\; =\frac{30}{49} \end{gathered}[/tex]

Therefore, the probability of the event that at least one of the balls is white is 30/49


A triangle, ABC, has angle measures of 45°, 45°, and 90° and two congruent (equal) sides. How would this triangle be classified?
O Isosceles acute
O Scalene acute
O Scalene right
O Isosceles right

Answers

Answer:

  (d)  Isosceles right

Step-by-step explanation:

You want to know the classification of a triangle with angles 45°-45°-90°.

Classification

The largest angle is 90°, a right angle. Any triangle containing a right angle is a right triangle.

Two of the angles have the same measure. This means the triangle is an isosceles triangle.

The triangle is classified as an isosceles right triangle.

__

Additional comment

A triangle whose largest angle is less than 90° is an acute triangle. If the largest angle is greater than 90°, it is an obtuse triangle.

If no sides are the same length, the triangle is scalene. If two sides are the same length, the triangle is isosceles. If all three sides are the same length, it is an equilateral triangle.

Larger angles are opposite longer sides, so if angles are the same, their opposite sides are the same.

It can be worthwhile to remember that the side length ratios in an isosceles right triangle are 1 : 1 : √2.

Help i need an answer.

Answers

B and C must be parallel

i need help with this asap please

Answers

Answer:

Given that,

[tex]2+(-8)+32+(-128)+.\ldots_{}[/tex]

To find the sum of the first 5 terms.

First, to find the first 5 terms of the given sequence.

The given sequence is 2,-8,32,-128,...

It follows geometric series with initial term 2, and common ratio as -4

The explicit formula of the given sequence is,

[tex]t_n=2(-4)^{n-1}_{}_{}[/tex]

To find the 5th term of the sequence,

Put n=5 in the above equation we get,

[tex]t_5=2(-4)^{5-1}[/tex][tex]t_5=2(-4)^4[/tex][tex]t_5=2(256)[/tex][tex]t_5=512[/tex]

Since common ratio is less than 1, we get the sum of the series formula as,

[tex]S_n=\frac{a(1-r^n)}{1-r}[/tex]

Substituting the values we get,

[tex]S_5=\frac{2(1-(-4)^5)}{1+4}[/tex][tex]=\frac{2(1+1024)}{5}[/tex][tex]=\frac{2(1025)}{5}[/tex][tex]=2(205)[/tex][tex]=410[/tex]

The sum of the first 5 terms of the given series is 410.

Answer is: option B: 410

There are 2.54 centimeters in 1 Inch. There are 100 centimeters in 1 meter. To the nearest inch, how many inches are in 7 meters? Enter the answer in the box. inches

Answers

Based on the given equivalences, you have:

7 m = 7(100 cm) = 700 cm

700 cm = 700 (2.54 in) = 1,778 in

Hence, there are 1,778 inches in 7 meters

Find the area of a triangular window with the given base and height base = 10 ft height =7 ft

Answers

The area of a triangular shape is given by the following formula

[tex]A=\frac{bh}{2}[/tex]

where,

base, b = 10 ft

height, h = 7 ft

therefore,

[tex]A=\frac{10\cdot7}{2}=\frac{70}{2}=35[/tex]

thus, the answer is 35 ft^2

Study the spinner below. If the wedges 2 and 5 are twice the area of the otherwedges what is the likelihood of landing on 4 or 6?23416/5A.NIO.B.A-OC.100oD. There is not enough information provided.

Answers

As we can see, there are 8 parts in our circle but 4 parts belong to wedge 2 and 5. Then, the probability of landing on 4 or 6 is given by

[tex]P(4\cup5)=P(4)+P(5)[/tex]

because part 4 and part 5 are disjoint sets. Since there are 8 parts in our circle, we have

[tex]\begin{gathered} P(4\cup5)=P(4)+p(5) \\ P(4\cup5)=\frac{1}{8}+\frac{1}{8} \end{gathered}[/tex]

which gives

[tex]P(4\cup5)=\frac{2}{8}=\frac{1}{4}[/tex]

which corresponds to option B

I need help with part b.

Answers

log ( √2.86668684 ) is value of logarithm 18 .

What is logarithm used for in math?

In order to get another number, a number must be raised to a certain power, which is known as a logarithm (see Section 3 of this Math Review for more about exponents). For instance, the base ten logarithm of 100 is 2, since ten multiplied by two yields the number 100: log 100 = 2.The answer is found using a logarithm (or log).

[tex]log_{b} \sqrt{18}[/tex]  = [tex]log_{b} = log_{b} \sqrt{15} + log_{b} \sqrt{3}[/tex]

                 = log( √15 + √3 )

                = log ( √1.0986 * 1.6094  + √1.0986 )

                = log ( √1.76808684 + √1.0986 )

                = log ( √2.86668684 )

Learn more about logarithm

brainly.com/question/28346542

#SPJ13

Question 1. Find the center and radius of the circunscribed circle.

Answers

In order to find the center of the circunscribed circle, we can use the midpoint theorem because the center point is in the middle of any two vertices

that is, if we take points (9,23) and (8,16) the midpoint C is given as

[tex]C=(\frac{8+9}{2},\frac{16+23}{2})[/tex]

which gives

[tex]C=(8.5,19.5)[/tex]

So the center of the circle is the point (8.5,19.5)

On the other hand, the radius is equal to the distance from any vertex to the center. If we take the vertex (8,16), we get

[tex]r=\sqrt[]{(8.5-8)^2+(19.5-16)^2}[/tex]

which gives

[tex]\begin{gathered} r=\sqrt[]{0.5^2+3.5^2} \\ r=\sqrt[]{0.25+12.25} \\ r=\sqrt[]{12.5} \\ r=3.5355 \end{gathered}[/tex]

so, the radius measure 3.54 units.

Now, lets prove that the answer are correct. In order to do that, we can choose the other vertices and apply the same procedure as above.

So the vertices are (5,20) and (12,19). Again, the center is the midpoint between these points and is given as

[tex]\begin{gathered} C=(\frac{5+12}{2},\frac{20+19}{2}) \\ C=(\frac{17}{2},\frac{39}{2}) \\ C=(8.5,19.5) \end{gathered}[/tex]

which is the same center as above.

Now, the distance from the center to vertec (5,20) is

[tex]\begin{gathered} r=\sqrt[]{(8.5-5)^2+(20-19.5)^2} \\ r=\sqrt[]{3.5^2+0.5^2} \\ r=\sqrt[]{12.25+0.25} \\ r=\sqrt[]{12.5} \\ r=3.5355 \end{gathered}[/tex]

which is the same radius obtained above. Then, the answers are correct.

What quadrant is 0 And. . -1 1/2 in or is it on a y- axis or x-axis.

Answers

The coordinate pair is between the third and fourth quadrants, on the y-axis.

In which quadrant is the coordinate point?

Remember that the quadrants are:

First quadrant: x > 0, y > 0.

Second quadrant: x < 0, y > 0.

Third quadrant: x < 0, y < 0.

Fourth quadrant: x > 0, y < 0.

In this case our coordinate pair is (0, -1/2).

So it will be between the third and fourth quadrants.

And yes, because one of the variables is zero, it is on the y-axis (just between the two quadrants).

Learn more about coordinate pairs.

https://brainly.com/question/17206319

#SPJ1

When you mix two colors of paint in equivalent ratios, the resulting color is always the same. Complete the table as you answer the questions.

How many cups of yellow paint should you mix with 1 cup of blue paint to make the same shade of green? Explain or show your reasoning.

Make up a new pair of numbers that would make the same shade of green. Explain how you know they would make the same shade of green.

row 1

cups of blue paint

cups of yellow paint

row 2

2

10

row 3

1

5

row 4

3

15

What is the proportional relationship represented by this table?

What is the constant of proportionality? What does it represent?

Answers

By Calculating the Constant of Proportionality, we get,

1. 5 cups of yellow paint is required for 1 cup of blue paint.

2. For 4 cups of blue paint, 20 cups of yellow paint is required.

   For 5 cups of blue paint, 25 cups of yellow paint are required.

3. Cups of Yellow Paints = 5* Cups of Blue Paints

4. The constant of Proportionality is 5.

Let the cups of blue paint = x

and cups of yellow paint = y

From the table, we can infer that to make the same shade of green, we need to mix 5 cups of yellow color with 1 cup of blue.

We have, To make the same shade of green,

we need to mix 5 cups of yellow color with 1 cup of blue.

we need to mix 10 cups of yellow color with 2 cups of blue.

we need to mix 15 cups of yellow color with 3 cups of blue.

So, here we can see a relationship between the two colors, blue (x) and green(y)  

Let k be the constant of proportionality, then, we have :

10 = k *2

k =[tex]\frac{10}{2} = 5[/tex]

Hence, For one cup of blue paint, we need 5 cups of yellow paint to make the same shade of green.

And the equation of the same is y =kx, that is y =5x....equation(1)

To make new pair of numbers that would make the same shade of green.

We can use the equation 1,

for x = 4, we need y = 20

foe x =5, we need y = 25

To read more about the constant of proportionality, visit https://brainly.com/question/22620356

#SPJ1

The nutty professor sells cashews for $6.20 per pound and Brazil nuts for $4.30 per pound. How much of each type should be used to make a 32 pound mixture that sells for $5.37 per pound?

Answers

Let's call C the weight of the Cashews and B the weight of the Brazil nuts.

Since the professor wants to sel a 32 pound mixture, we have the following equation:

[tex]C+B=32.[/tex]

Now, since the price of the mixture will be $5.37 per pound, this means the price of the whole mixture will be

[tex]32\cdot5.37=171.84.[/tex]

This leads us to the followin equation:

[tex]6.20C+4.30B=171.84.[/tex]

Now we have a system of equations:

[tex]C+B=32[/tex][tex]6.20C+4.30B=171.84.[/tex]

To solve it, let's isolate one of the variables in the first equation by subtracting B from both sides of it:

[tex]C=32-B\text{.}[/tex]

Now, let's use this value of C in the second equation:

[tex]6.20(32-B)+4.30B=171.84,[/tex][tex]198.4-6.20B+4.30B=171.84,[/tex][tex]198.4-1.9B=171.84.[/tex]

To solve this equation, let's subtract 198.4 from both sides of it:

[tex]-1.9B=-26.56.[/tex]

Then, let's divide both sides by -1.9:

[tex]B\approx13.98.[/tex]

Using this value of B in the first equation will give us:

[tex]C=32-13.98=18.02.[/tex]

These would be the values of B and C round to two decimals. If we want, we can also write them as integer numbers. so the mix would need to have 18 pounds of cashews and 14 pounds of Brazil nuts.

Other Questions
What is the actual distance between these two cities in kilometers? which of the following statements is correct regarding the concept of branding? question 6 options: brands cannot be built through advertising, public relations, sponsorships, social media, or similar techniques. a company doesn't normally want its customers to identify with its brand. startups usually start with an established brand. a brand can be one of a company's most valuable assets. a brand can denote a negative impression of a company. Which is true about investments and risk? questions about the extent to which maladaptive habits learned in childhood can be overcome in adulthood are most directly relevant to the issue of: please choose the correct answer from the following choices, and then select the submit answer button. answer choices free will and determinism. stability and change. nature and nurture. continuity and stages. Are the Whiz Kid practices helpful or hurtful for Melody? Solve for A R= X (A+ B) emil company expects that its asset will be more useful during early years of its life than during later years. in addition, the company estimates that repair costs will increase over time. which method(s) may help equalize total expenses recognized over the service life of this asset? (select all that apply.) a nurse teaches a client with obesity about the various medication options for the treatment of obesity. which medication will the nurse include when teaching the client about the class of medications that stimulate noradrenergic receptors? abstract, concrete, countable/noncount nouns. 1. That autumn Kate ... she would join some fitness centretook decisiondecidedmade her minddetermined2. Her working day in the office ... seven hourslastedprolongedpassedhad3. Practically all the time she had to sit ... the computerbeforebehindeveralongside4. Naturally she began to feel some pains in the back and realized she needed ...the exerciseany exercisesome exercisean exercise5. And Kate began jogging. She tried to do it each morning but she was not very ... about itlazydevitedseriousstrong6. When the weather was bad, Kate ... her runningdelayedput awaypostponedleft7. When she had a lot of work to do, she simply skipped ... that day :an activitythe trainninga classthe lesson8. Very soon Kate understood she had to be more systematic and she set up a trainig ... :programmeprojectlineoutline9. Every Tuesday she had to run 3 kilometers, every Thursday not less than 2 kilometers and every Saturday as far as possible. Once a week she visited the fitness centre. By the beginning of November she was jogging in rain and wind and once she did it ... in snow. She got used to itelseeventhusthough10. By the end of the year Kate had ... changed her lifestylerarelypossiblyreallyfairly Quadrilateral ABCD has coordinated a(-4,3) b(-3,6) c(0,8) d(-2,5). Prove that quadrilateral ABCD is a parallelogram and explain whether quadrilateral ABCD is a rectangle or not. Sarah got a $60 gift card to Walmart. She uses the gift card to buy orange juice for $4.50. She also wants to use the gift card to buy shirts. Each shirt costs $2.29. Write an inequality to describe this situation, where n is the number of shirts Sarah wants to buy?Please answer quick, will give 18 points What is the equation of a line that passes through (8,-5) and is parallel to the graphed line? In a linear graph line diagram, A line passes through (minus 4, minus 6) and (8, 3) which intersects the x-axis at 4 units and the y-axis at minus 3 units. common errors for Rational Numbers. or each ordered pair, determine whety=5x-315x-3y=9(2,7)(-1, 8)(3,4)(0,-9) Determine algebraically ifthe function is even, odd, or neither.f(x)=3x-2 Use the negative fractions -4/5 and -5/6PART AFind the decimal equivalents for each fraction.PART BWhich is a repeating decimal? Which digit isrepeating? the fundamental purpose of an organization's mission statement is to: a. define the operational structure of the organization. b. generate good public relations for the organization. c. define the organization's purpose in society. d. define the functional areas required by the organization. Show whether or not each pair of triangles are similar, if possible. Justify your answer, and write a similarity statement when the triangles are similar. Write your answer as a fraction or as a whole or mixed number 3 7/10 * 4 5/6